Jump to content

nix85

Members
  • Posts

    20
  • Joined

  • Last visited

Recent Profile Visitors

The recent visitors block is disabled and is not being shown to other users.

nix85's Achievements

Quark

Quark (2/13)

-4

Reputation

  1. Well, that's not really achivable so not relevant, but of course, it would add to efficiency even more. As for "relevant mathematics".. For example if two unbalanced masses oscillate within 180° (in opposite direction to cancel back torque) as part of the same system, we must take mass and radius of the spinning masses and average speed to get average force according to F = mv²/r. We must take into account only the x component of the forces (pulling the system forward) and that x component varies from 0 (when pendulums are 180° apart) and rises by sqare of velocity to max (when they overlap in front position). That average x-component of the force is constant pure unidirectional acceleration. Few demonstrations of the principle.
  2. What a foolishness. Weak minded always depend on "peer reviews" and so called "authorities". Lack of intelligence makes you desperately seek their approval and prevents you from seeing the simple facts. 4th LAW i defined simply means that person floating in space that swings his arms in front of him will accelerate forward as if pushed from behind, thereby violating noted "laws", we cannot call them laws cause they apply in very narrow set of conditions only. THERE IS NO CONSERVATION OF MOMENTUM THERE IS NO CONSERVATION OF ENERGY If there were this phenomena could not exist. As long as the pendulum in the picture below keeps oscillating in the yellow area or spinning 360° but slowing down in the red part - inclination of the cart irrelevant), it will keep accelerating forward. Superposition is also not a law, but a way to explain the observed effect, for example cancelation of magnetic fields on the outside of two opposing conductors. If it was really a law, it would also apply to two opposing permanent magnets, but we see no such cancelation, we only observe compression of the fields inbetween and APPARENT cancelation in exact middle between them, where two opposing forces are balanced. They do NOT cancel out as in example of two opposing conductors, why is this so should be further examined.
  3. That's a foolish statement as i am talking about a simple undenyable physical LAW. All of you 4 smartasses who gave me negative - reputation obviously don't like the truth.
  4. Zero problem. Only problem is in heads of those who see universe as a closed system. BTW even Einstein said rotating reference frames are non-inertial, in them virtual forces act which do not obey classical laws of conservation.
  5. Indeed. Which is a stone age approach to "antigravity" - of course has nothing to do with gravity (which is another subject i could write a bible about, but you have no need to know). I once posted this to NASA forum like 5-6 years ago, i got response "there is no free energy here" and instant ban.
  6. That's assumed,I don't know if Newton had a 4th law or not, i named this 4th law as it is an extension of his 3 laws. Still no deniers? You can at least try to attack.
  7. I think KNOWLEDGE carries way more weight than ignorance. Anyone dares to deny the 4th LAW as i defined it?
  8. It's got 12 upvotes at quora. Scroll down to Nikša. https://www.quora.com/What-is-Newton’s-fourth-law-of-motion-which-was-a-hidden-mystery
  9. Well, when you realize the reality and importance of the phenomena, you might. I'll give you a hint, this means you can literally "swim" in space. Of course assuming you swing arms in opposing directions to cancel the back torque. It also makes the death of Matt Kowalski VERY ironic (or just plain stupid).
  10. You don't have to measure inside the magnet to know that field is not able to penetrate the magnet, as it will affect the field on the opposing side. This is easy to test by putting a small magnet inside a stronger magnet's field and measuring the field on the "shaded" pole of the smaller magnet. Field will get compressed by the flux of the bigger magnet but will not let it pass through up to a certain point when it will "break down" just like dielectric breaking down under high voltage. From that point on smaller magnet is remagnetized in opposite direction. Yes i do, but you clearly do not. You are missing the point. Superposition may be used to describe cancelation of emission from two conductors carrying alternating current in opposite direction, but if we take two permanent magnets opposing each other and measure the exact middle between them, the fields will be intact (altho deformed), but probe will show 0 magnetism. So we have apparent cancelation, but in reality what we have is point at the boundary of the two repelling fields in an ideal, hypothetical environment, balance between opposing forces, not cancelation of the fields. Also superposition can be violated for example in a transformer, the iron core does not obey superposition when it has reached saturation. Point is that opposing magnetic fields do not penetrate each other altho we may observe effect as if they do. Of course it penetrates the shield. Flux follows the path of least reluctance. This is irreleavant to my point that fields of opposing magnets can not penetrate each other. You rather have PtsD. One of your pearls "waves are not vectors". What circus do you come from. BTW you are talking to a person who defined the 4th LAW OF MOTION, recognized at Quora and veryfied by independed sources world-wide (if it's not self-evident). Simple yet still denied phenomena that violates conservation of momentum and thus conservation of energy. Quote: "4th LAW OF MOTION simply states that asymmetrical centrifugal force of unbalanced masses converts into linear acceleration of the system. In other words, it means that unbalanced mass oscillating within 180° or less, or spinning with varying speed, accelerates the whole system unidirectionally without interacting with any outside medium apart for the space itself. " I got more knowledge in a dirt under my nail than you got in your skull. PS I hope you will deny the 4th LAW, i'm really looking forward to that. ;D
  11. Yes it does if we are talking permanent magnets. Opposing field cannot penetrate the magnet except by remagnetizing it in opposite direction. You are clearly unable to understand that opposing magnetic fields never overlap, we add vectors to predict the resultant field which always has a single value for a certain point in space, vector, scalar or tensor-wise. It is just a way of calculating things, not what really happens. Your confusion makes you see contradiction where there is none. "Waves aren't vectors" - needless to say who doesn't understands his sources. #facepalm Go back to first grade. To repeat once again one of basic laws of magnetism. "LINES OF MAGNETIC FLUX NEVER INTERSECT"
  12. That is exactly what i said, if one magnet is strong enough it will remagnetize the weaker (electro)magnet in opposite direction but that is not superposition but neutralization of the weaker field. You might think of it in a way that weaker field will oppose the stronger field to a certain point only. You seem to be under delusion that two opposing magnetic fields can overlap each other. In attraction they merge, in opposition they divert, NEVER overlap. Waves are not vectors? Waves have magnitude and direction, so they are vectors. Magnetic field is technically not a wave but they have direction and amplitude (field density), so they add as vectors and this is not a question. http://www.emfs.info/what/adding/ You can EFFECTIVELY cancel the magnetic field, for example two similar conductors in opposite directions "cancel out" their fields. But they do not REALLY cancel, they reinforce inbetween the wires and cancel on the outside. What really happens is: "Between the wires, the magnetic fields from the two wires point in the same direction so the resulting or total magnetic field will be large. Outside the wires, the two magnetic fields from the two currents are in opposite directions so they will tend to cancel each other and the resulting or total magnetic field will be small." Quote from http://www.ux1.eiu.edu/~cfadd/1360/30MagSrcs/HmwkSol.html You can also place two magnets facing with same pole and you will create a spot where fields are "canceled". Well, not really. Only apparently. In Helmholtz coil however two currents are in same directions so they add up creating a uniform field in the middle. In the very center aka bloch wall point of no magnetism is achieved. It matters not what you believe, facts are facts. My sources are good. Magnetic fields do NOT superimpose, only merge or divert. In this video i demonstrate that in case of ring magnets, same poles can ATTRACT (with quite force).
  13. 1. Like i said we can only imagine them intersecting, draw vectors and resultant vectors, but they never do. 2. Indeed! That is a very good point as in fact, whole creation is but one big magnetic field, one big electron-antielectron pair. Video is not important, i linked it to show how very strong field can magnetize neodymium, which is relevant to the subject. It would be even better if small magnets were already magnetized and forced into the field opposing it, we can assume they would get remagnetized in opposite direction. Yes, magnetic fields don't obey superposition. We can't speak of superposition of magnetic fields as of EM waves. There is no constructive or destructive interference with magnetic fields. When in attraction they add-up, that is, increase density of the field, when in opposition they bend away from each other. They do not superimpose. This is addressed in this article. http://subato.blogspot.com/2011/03/how-can-magnetic-fields-interfere.html
×
×
  • Create New...

Important Information

We have placed cookies on your device to help make this website better. You can adjust your cookie settings, otherwise we'll assume you're okay to continue.